Write in point-slope form an equation of the line that passes through the point (8, 9) with slope 7.

Answers

Answer 1

Answer:

y=7x-47

Step-by-step explanation:

[tex](8,9)=(x1,y1) \\ m=7 \\ y−y1=m(x−x1) \\ y−9=7(x−8)

[/tex]

[tex]y - 9 = 7x - 56 \\ y = 7x - 56 + 9 \\ y = 7x - 47[/tex]


Related Questions

Geometry, please answer question ASAP

Answers

Answer:

D. 101

Step-by-step explanation:

(25x+1)+(25x-2)+(20x-1)+82=360

70x=360-1+2+1-82

70x=280

x=4

A=25x+1=101

Answer:

m∠A = 101*

Step-by-step explanation:

m∠A = (25x + 1)*

m∠B = 82*

m∠C = (25x - 2)*

m∠D = (20x - 1)*

Find m∠A. Key: solve for x

(25x + 1) + 82 + (25x - 2) + (20x - 1) = 360*

Subtract 82 from both sides

(25x + 1) + (25x - 2) + (20x - 1) = 278*


x = 4

Now substitute x for 4 in A measure

(25x + 1)

(25(4) + 1)

(100 + 1)

101

m∠A = 101*

I REALLY NEED HELLP with these 3 questions PLLZZZZ!!!!

Answers

Answer:

Below

Step-by-step explanation:

6)

The sum that we have is 85 +99

We want to express it as the product of a whole number thar is greater than 1 and a sum of two whole numbers.

Notice that: 85 = 84+1

● 85 + 99 = 84 + 1 + 99 = 84 + 100

84 and 100 are even numbers so we can factor using 2.

● 84 + 100 = 2(42 +50)

2 is greater than one and 42+50 is the sum of two whole numbers so all the conditions are satisfied.

■■■■■■■■■■■■■■■■■■■■■■■■■■

7)

Dasha go on business trips every 9 months while Charlie go every 6 months.

They came back at the same time.

So Charlie has to wait 6 months before going and Dasha nine months.

Dasha will be alone home for 3 months so she doesn't need to hire someone.

Here is what happens:

● Both Dasha and Charlie are home.

● After 6 months Charlie go and Dasha is at home

● after 3 months Dasha goes also and Charlie is home

● after 3 months charlie go and Dasha is home

● after 3 months both are home.

● aftet 3 months they both go

So the period is:

● 6+3+3+3+3 = 18

So after 18 months they should hire someone.

The picture below makes the understanding easier. ( x is Charlie and y is Dasha)

■■■■■■■■■■■■■■■■■■■■■■■■■■

8)

Pime factorisation

● 96÷2= 48

● 48÷2= 24

● 24 ÷ 2 = 12

● 12 ÷ 2 = 6

● 6÷2 = 3

● 3÷3 = 1

=> 96 = 2 × 2 ×2×2×2 ×3 =2^5 ×3

● 80÷ 2 = 40

● 40 ÷ 2 = 20

● 20÷2 = 10

● 10÷2 = 5

● 5÷5 = 1

=> 80 = 2×2×2×2×5 = 2^4 × 5

So the GCF is 2^4 wich is 16

He can make 16 party

● 80÷16 = 5

There will be 5 boxes of raisin in each one

● 96÷16 = 6

There will be 6 pencils in each party

Becky's ship is 43 miles west all the harbor.
Clyde's yacht is a5 miles north from Beery. How
far is Clyde from the Harbor? Show your work.
С
x= Harbor
25
B.
43

Answers

Answer:

Clyde is 49.74 away from the harbor

Step-by-step explanation:

Here in this question, we are interested in knowing the distance of Clyde from the harbor.

The key to answering this question is having a correct diagrammatic representation. Please check attachment for this.

We can see we have the formation of a right angled triangle with the distance between Clyde’s ship and the harbor the hypotenuse.

To calculate the distance between the two, we shall employ the use of Pythagoras’ theorem which states that the square of the hypotenuse is equal the sum of the squares of the two other sides.

Let’s call the distance we want to calculate h.

Mathematically;

h^2 = 25^2 + 43^2

h^2 = 625 + 1849

h^2 = 2474

h = √2474

h = 49.74 miles

PLEASE HELP WILL MARK BRAINLIEST

Answers

Answer:

18 m

Step-by-step explanation:

Answer:

18m

Step-by-step explanation:

GIVING BRAINLIEST TO THE FIRST PERSON TO ANSWER!

Three stores have the same tablet computer on sale. The regular price of the tablet is $150. Store A is offering the tablet on sale at 15% off the regular price. Store B is offering a $25 coupon to be deducted from the regular price. Store C is offering a rebate of $20.00 to purchasers. Store D has the tablet on sale for $120.00. Which store is offering the tablet at the lowest cost?

A. Store A

B. Store B

C. Store C

D. Store D

Show ALL work please! <3

Answers

Answer: Store D

Step-by-step explanation:

Store A - 15% off:

$150 times 15/100 = 22.5

$150 - $22.50 = $127.50 so the price at store A is $127.50

Store B - $25 coupon

$150 - $25 = $125 so the price at store B is $125

Store C - $20 rebate

$150 - $20 = $130 so the price at store B is $130

Store D - $120

$120 is the lowest so the answer is Store D

Answer:

[tex]\Large \boxed{\mathrm{Store \ D}}[/tex]

Step-by-step explanation:

Store A is offering the tablet on sale at 15% off the regular price.

150 × (1 - 15%) = 127.5

Store B is offering a $25 coupon to be deducted from the regular price.

150 - 25 = 125

Store C is offering a rebate of $20.00 to purchasers.

150 - 20 = 130

Store D has the tablet on sale for $120.00.

Store D is offering the tablet at the lowest cost.

convert -1.2i - 1.6j to polar cordinates

Answers

Answer:

Step-by-step explanation:

z=xi+yj

x=-1.2=r cosθ (say)

y=-1.6=r sinθ(say)

square and add

r²cos²θ+r² sin ² θ=(-1.2)²+(-1.6)²=1.44+2.56=4

r²=4

r=2

divide

tanθ=y/x=-1.6/(-1.2)=16/12=4/3

θ=tan ^{-1}(4/3)≈53.13°

so polar coordinates are (2,53.13°)

BRAINIEST!!! only answer if you know and can give an explanation, will report for non-sense answers

Answers

Answer:

Below

Step-by-step explanation:

For a given shape to be a rhombus, it should satisfy these conditions:

● The diagonals should intercept each others in the midpoint.

● The diagonals should be perpendicular.

● The sides should have the same length.

We will prove the conditions one by one.

■■■■■■■■■■■■■■■■■■■■■■■■■■

Let's prove that the diagonals are perpendicular:

To do that we will write express them as vectors

The two vectors are EG and DF.

The coordinates of the four points are:

● E(0,2c)

● G (0,0)

● F (a+b, c)

● D (-a-b, c)

Now the coordinates of the vectors:

● EG (0-0,0-2c) => EG(0,-2c)

● DF ( a+b-(-a-b),c-c) => DF (2a+2b,0)

For the diagonals to be perpendicular the scalar product of EG and DF should be null.

● EG.DF = 0*(2a+2b)+(-2c)*0 = 0

So the diagonals are perpendicular.

■■■■■■■■■■■■■■■■■■■■■■■■■■

Let's prove that the diagonals intercept each others at the midpoints.

The diagonals EG and DF should have the same midpoint.

● The midpoint of EG:

We can figure it out without calculations. Since G is located at (0,0) and E at (0,2c) then the distance between E and G is 2c.

Then the midpoint is located at (0,c)

● The midpoint of DF:

We will use the midpoint formula.

The coordinates of the two points are:

● F (a+b,c)

● D(-a-b,c)

Let M be the midpoint of DF

●M( (a+b-a-b,c+c)

● M (0,2c)

So EG and DF have the same midpoint.

■■■■■■■■■■■■■■■■■■■■■■■■■■

There is no need to prove the last condition, since the two above guarante it.

But we can prove it using the pythagorian theorem.

Two lines cross to make the letter x. The letter is 5 inches wide. What is the angle where the lines cross

Answers

Answer:

24

Step-by-step explanation:

trudt

How to do this question plz.
plz work out for me in your notebook or sheet if you can plz the question ​so I can understand more plzz

Answers

Answer:

[tex]3\pi[/tex]

Step-by-step explanation:

The circumference of a circle is [tex]2\pi r[/tex].

If we want to find the circumference of this semi-circle, we can find the circumference if it was a whole circle then divide by 2.

[tex]2 \cdot \pi \cdot r\\2 \cdot \pi \cdot 3\\6 \cdot \pi\\ 6\pi[/tex]

Now we know the circumference of the whole circle.

To find the circumference of half the circle we divide by 2.

[tex]6\pi \div 2 = 3\pi[/tex]

Hope this helped!

Find the length of side x in simplest radical form with a rational denominator.

Answers

Answer:

x = 3

Step-by-step explanation:

Since this is a right triangle, we can use trig functions

tan theta = opp side / adj side

tan 30 = sqrt(3) / x

x tan 30 = sqrt(3)

x = sqrt(3) / tan 30

x =sqrt(3) / (1 / sqrt(3))

x = sqrt(3) * sqrt(3)

x = 3

Determine if the expression 464 +c+ cº is a polynomial or not. If it is a polynomial,
state the type and degree of the polynomial.

Answers

Answer:

I wish I could help but I'm not sure

Help mee whats the answerr :( Q: The midpoint of the line segment with the coordinates (9, -8) and (-2, 1) is _

Answers

Answer:

(3.5, - 3.5)

Step-by-step explanation:

Using the Midpoint theorem, we have

x=(9-2)/2=7/2=3.5

y=(-8+1)/2=-7/2=-3.5

Please help me with these questions please.

Answers

Answer:

see image

Step-by-step explanation:

The population of fruit flies tripled every day and the population today is 200 fruit flies.
a) Write an equation modeling the growth in the population of fruit flies.
b) What was the population of fruit flies 3 days ago?

Answers

Answer:

Equation = 600nNumber of fruit flies after 3 days = 1800 fruit flies

Step-by-step explanation:

Given:

Number of fruit flies = 200

Rate of growth = 3 times per day

Find:

A . Equation

B . Number of fruit flies after 3 days

Computation:

A . Equation = 3n(200)

Equation = 600n

B. Number of fruit flies after 3 days

Number of fruit flies after 3 days = 600(3).

Number of fruit flies after 3 days = 1800 fruit flies


Is a – b a positive number or a negative number? How do you know?
b b
a
1
Please help I will give you Brainlyest

Answers

Answer:

A positive number.

Any negative subtracted from a positive will always be positive.

Answer:

a-b will be and will always be a POSITIVE number.

This is because a is positive, and b is negative.

However, when you subtract a negative number, you are actually just adding it.

So, a - (-b) = a+b.

This is why, it will always be positive.

Let me know if this helps!

Which data set matches the box-and-whisker plot?
A) 12 13 15 19 23 23 25 26.5 28 30
B) 15 13 19 21 23 24 27 29 32
C) 11 31 13 15 19 21 21 25 27 29 31
D) 11 13 15 19 23 23 24 26.5 28 33​

Answers

Answer:

D) 11 13 15 19 23 23 24 26.5 28 33​

Step-by-step explanation:

The box-and-whisker plot displayed above has the following key values that we can use to identify which of the given data set it matches. It has:

Minimum value = 11

Q1 = 15

Median = 23

Q3 = 26

Maximum value = 33

From the options given, using just the max and min value, we can conclude that the data set in option D matches the box plot.

The data set in option D has a minimum value of 11, and a maximum value of 33.

A piece of aluminum occupies a volume of 12.7 milliliters and weighs 87.3 grams. What is its density of the aluminum rounded to the nearest hundredth? Only enter numerical values, which can include a decimal point.

Answers

Answer:

6.87 g/mL

Step-by-step explanation:

The density of an object can be found by dividing the mass by the volume.

[tex]density=\frac{mass}{volume}\\\\ d=\frac{m}{ v}[/tex]

We know that the aluminum occupies a volume of 12.7 milliliters and weighs 87.3 grams. Therefore, the mass is 87.3 g and the volume is 12.7 mL.

[tex]m= 87.3 g\\\\v=12.7 mL[/tex]

Substitute the values into the formula.

[tex]d= \frac{87.3 g}{12.7 mL}[/tex]

Divide 87.3 g by 12.7 mL

[tex]d=6.87401575 g/mL[/tex]

Round to the nearest hundredth. The 4 in the thousandth place tells us to leave the 7 in the hundredth place.

[tex]d= 6.87 g/mL[/tex]

The density of the aluminum is about 6.87 grams per milliliter.

You move down 5 units and up 5 units. You end at (1,3). Where did you start?

Answers

Answer:

(1, 3)

Step-by-step explanation:

Since up and down are opposites, the point didn't move at all.

I hope this helps!

pls ❤ and mark brainliest pls!


Rearrange the equation below to identify the variables a, b and c.
0 = 4x – 5x2​

Answers

Answer:

a = -5x

b = 4

c = 0

Step-by-step explanation:

binomial equation is written in form of

[tex]ax^2 + bx + c = 0[/tex]

given equation

[tex]0 = 4x - 5x^2[/tex]

rearranging it we have

[tex]0 = - 5x^2 + 4x + 0[/tex]

comparing this equation with [tex]ax^2 + bx + c = 0[/tex]

comparing coefficient of variables  with same power.

a = -5x

b = 4

c = 0

What is the solution to the equation 1 over the square root of 8 = 4^(m + 2)? A) m = − 15 over 4 B) m = − 11 over 4 C) m = 5 over 4 D) m = 9 over 4

Answers

Answer:

B) m = -11/4

Step-by-step explanation:

What is the solution to the equation 1/sqrt(8) = 4^(m+2)

1 over the square root of 8 = 4^(m + 2)?

A) m = − 15 / 4

B) m = − 11 / 4

C) m = 5 / 4

D) m = 9 / 4

1/sqrt(8) = 4^(m+2)   apply law of exponents

1/2^(3/2) = (2^2)^(2m+4)

2^(-3/2) = (2)^(2m+4)

-3/2 = 2m+4

2m = -11/2

m = -11/4

the number ten is raised to a power between 0 and 1. The answer has to be between which two numbers?

Answers

Any number raised to 0 is equal to 1
And any number raised to 1 is the same number

   Value of [tex]10^x[/tex] will be between 1 and 10.

 Given in the question,

A number [tex]10^x[/tex] where, 0 < x < 1

If we have to find the range of the value of the number, substitute x = 0 and 1

[tex]10^0=1[/tex]

[tex]10^1=10[/tex]

  Therefore, value of [tex]10^x[/tex] will vary between 1 and 10.

Learn more,

https://brainly.com/question/19465964

Steel chain costs $0.75 per foot from Company
X, which provides free shipping. The same
steel chain costs $1.20 per yard at Company Y,
which charges a $120.00 shipping fee for all
orders. How much more does Company
Y charge, in dollars, than company X for 300
feet of steel chains?

Answers

Answer:

Company Y charges 255 dollars more then Company X :)

Step-by-step explanation:

Company X: 0.75 * 300 = 225 + 0 shipping

Company Y: 1.20 * 300 = 360 + 120 shipping = 480

480 - 225 = 255

a bottle is completely filled with olive oil the mass of the bottle is 500 grams if the density of the oil os 0.92 grams per milliliter what is the volume of the bottle to the nearest milliliter?

Answers

Answer:

543.48 millimetre

Step-by-step explanation:

mass/density = volume

500 grams / 0.92 grams per millimetre = 543.48

Answer:

volume = 543.478 cm³

Step-by-step explanation:

Density = mass / volume

0.92g/ml = 500g / volume

volume (0.92g/ml) = 500g

volume = 500g / (0.92g/ml)

volume = 543.478 ml    (aprox. to the nearest mililiter)

1 ml = 1cm³

543.478ml = 543.478 cm³

what’s the equation

Answers

Answer:

y = 4. [tex]3^{x}[/tex]

Step-by-step explanation:

For an exponential function in the form

y = a[tex]b^{x}[/tex]

Use ordered pairs from the table to find a and b

Using (0, 4 ), then

4 = a [tex]b^{0}[/tex] ( note [tex]b^{0}[/tex] = 1 ), thus

a = 4

y = 4 [tex]b^{x}[/tex]

Using (1, 12 ), then

12 = 4 [tex]b^{1}[/tex] = 4b ( divide both sides by 4 )

b = 3

Thus

y = 4 . [tex]3^{x}[/tex] ← exponential equation

If there are:
6 red marbles
10 yellow marbles
5 green marbles
and 1 blue marble
What is the probability of picking 1 red marble and 1 green marble?

(Once a marble is chosen it IS put back into the box).

(Write your answer in the form of a decimal, round to 2 places).

Answers

Answer:

[tex]probability = \frac{1}{2}[/tex]

Step-by-step explanation:

Probability is the chance of an event happening. It can simply be defined by the following formula,

[tex]probability = \frac{desired\ outcomes}{total\ outcomes}[/tex]

The total outcome is all of the possible outcomes or the sum of all of the types of marbles. The desired outcomes are the outcomes that one wants or the sum of the number of wanted marbles,

[tex]total\ outcomes = red+yellow+green+blue\\=6+10+5+1\\=22[/tex]

[tex]desried\ outcomes= red+green\\= 6 + 5\\=11[/tex]

Substitute into the formula to find the probability of an event:

[tex]probability = \frac{desired\ outcomes}{total\ outcomes}[/tex]

[tex]probability = \frac{11}{22}[/tex]

Simplify,

[tex]probability = \frac{11}{22}[/tex]

[tex]probability = \frac{1}{2}[/tex]

I need help ASAP!! Please explain how to solve the problem

Answers

Answer:

B=.5*8*6=.5*48=24

h = 8

P=10+6+8=24

Surface area =2*24+24*8=48+192=240

Step-by-step explanation:

The base is a triangle, and the area of a triangle is 1/2 base (or width) times height (or length). In this case, base is 8 and height is 6, so the area can be found either by multiplying .5*8*6 or by dividing 8*6 by 2, either of which will give you the same answer of B=24.

Height is given, h=8.

Perimeter is simply the sum of the sides, which are given, so the perimeter is 8+6+10=24.

The formula for surface area is given, so all you need to do is plug in your previous answers and you've got it!

Simplify the expression

Answers

3^5 divided by 3^3 is 9

Answer:

B

Step-by-step explanation:

[tex] \frac{{3}^{5} }{ {3}^{3} } = {3}^{5 - 3} = {3}^{2} = 9 [/tex]

If Eric can paint $3$ cars in $4$ hours and $2$ trucks in $5$ hours, then how long, in hours, would it take him to paint $4$ cars and a truck? Express your answer as a common fraction.

Answers

Answer:

47/6

Step-by-step explanation:

Given that :

Time taken to paint 3 cars = 4 hours

Time taken to paint 2 trucks = 5 hours

How long will it take him to paint 4 cars and a truck

If 3 cars = 4 hours ;

Then ;

1 car = (4/3)hours

If 2 trucks = 5 hours

Then;

1 truck = (5/2) hours = 2 1/2 hours

Time required To paint 4 cars :

4 × (4/3) = 16/3 hours

Time required to paint 1 truck :5/2 hours

Total time required :

(16/3 + 5/2) = (32 + 15) / 6 = 47/6

Simplify square roots
Simplify.
Remove all perfect squares from inside the square root.
12


Answers

Answer:

[tex]2\sqrt{3}[/tex]

Step-by-step explanation:

[tex]\sqrt{12}[/tex]

[tex]\sqrt{(4)(3)}[/tex]

square root of 4 is 2

[tex]2\sqrt{3}[/tex]

the 3 stays inside

A rectangular bathroom mirror has a perimeter of 22 feet. Its area is 18 square feet. What are the dimensions of the mirror?

Answers

Answer:

The dimensions of the mirror is 9 feet and 2 feet

Step-by-step explanation:

Keep in mind that the area and perimeter of a rectangle is:

Area - length x width

Perimeter - 2 (l + w)

List the factors of 18 (the area of the mirror):

1, 2, 3, 6, 9, 18

POSSIBLE DIMENSIONS OF MIRROR:

1 ft and 18 ft

Area = 18 ft^2

Perimeter = 38 feet

2 ft and 9 ft

Area = 18 ft^2

Perimeter = 22 feet

3 ft and 6 ft

Perimeter = 18 feet

The rectangle with dimensions 2 feet and 9 feet corresponds with the area and perimeter of the mirror mentioned. So those are the correct dimensions.

Hope these helps!

Other Questions
Express using exponents and simplify any numerical coefficients. How much would you need to deposit in an account now in order to have $6,000 in the account in 8 years? Assume the account earns 6% interest compounded monthly. (could anyone do this whole problem out? Which event organised peace between Germany and the allied forces at the end of word war I ? A. the armistice B. the Spanish flu pandemic C. the signing of the Treaty of Versailles D. the signing of the Treaty of Brest-Litovsk PIOs should A. request additional assistance through the National Public Information Support Network. B. seek media release approval from the Communication Unit Leader. C. compile a go kit to allow for lengthy deployments to a long-term incident. D. report to the Finance/Admin Section Chief when arriving at an incident. A factory inspector found flaws in 3 out of 18 wooden boxes. What is the experimental probability that the next wooden box will be flawed?Write your answer as a fraction or whole number. a man bought a car for 8500 GH cedis and he later sold it at for 9500. find his percentage gain NEED HELP..... Select the polynomial that is a perfect square trinomial. 36x^2 4x + 16 16x^2 8x + 36 25x^2 + 9x + 4 4x^2 + 20x + 25 Graham Motors manufactures specialty tractors. It has two divisions: a Treactor Division and a Tire Division. The Tractor Division can use the tires produced by the Tire Division. The market price per tire is $75. The Tire Division has the following costs per tire:Direct material cost per tire: $15Conversion costs per tire: $3 (Assume the $3 includes only the variable portion of conversion costs.)Fixed manufacturing overhead cost for the year is expected to total $116,000. The Tire Division expects to manufacture 58,000 tires this year. The fixed manufacturing overhead per tire is $2 ($116,000 divided by 58,000 tires).Assume that the Tire Division has excess capacity, meaning that it can produce tires for the Tractor Division without giving up any of its current tire sales to outsiders. If Graham Motors has a negotiated transfer price policy, what is the lowest acceptable transfer price? What is the highest acceptable transfer price? The lowest acceptable transfer price is $_______, the Tire Division's ______________. The highest acceptable transfer price is $_______, the Tire Division's _____________. evaluate 5!+2!. Thank you! Which inequality is represented by the graph? What is acid rain? What chemicals make rain acidic, and how does it happen? How does acid rain affect humans? Are there any health problems associated with acid rain? What effect does acid rain have on trees and soil? What happens when lakes and aquatic systems become acidic? Are the aquatic systems in our area being affected by acid rain? If yes, to what extent? 3(q7)=27 need help plzz 1st peep gets brainlest Write an essay of 300 words including the following three parts: Describe the region of Central America, South America, and the Caribbean's most pressing environmental issue. Explain why you think the issue you selected presents the biggest problem. Discuss steps that can be taken by individuals, businesses and/or governments to reduce this threat. During a titration, a known concentration of _____ is added to a _____ of an unknown concentration g Mason Corporation had $1,150,000 in invested assets, sales of $1,228,000, income from operations amounting to $226,000, and a desired minimum rate of return of 12%. Round your answer to two decimal places. The investment turnover for Mason Corporation is a.0.85 b.1.07 c.1.28 d.1.60 Tatenda takes ttt seconds to mow a square meter of lawn and Ciara takes ccc seconds to mow a square meter of lawn. Tatenda mows 700700700 square meters of lawn per week and Ciara mows 750750750 square meters of lawn per week. Which expressions can we use to describe how many more seconds Tatenda spends than Ciara spends mowing lawns during 444 weeks? Choose 2 answers: Choose 2 answers: (Choice A) A 4(750c-700t)4(750c700t)4, left parenthesis, 750, c, minus, 700, t, right parenthesis (Choice B) B 3000c+2800t3000c+2800t3000, c, plus, 2800, t (Choice C) C 2800t-3000c2800t3000c2800, t, minus, 3000, c (Choice D) D 4(700t-750c)4(700t750c)4, left parenthesis, 700, t, minus, 750, c, right parenthesis (Choice E) E 4(700t+750c)4(700t+750c) What is the measure of m?6318nm ==[?] Read the following sentence.Industrialized nations are taking strict measures to reduce greenhouse gas emissionsWhich word could be substituted for taking, if the speaker wanted to express a negative viewpointimposingrecommendingconsideringadvising Sanders, a 62-year-old single individual, sold his principal residence for the net amount of $500,000 after all selling expenses. Sanders bought the house 15 years ago and has occupied it until it sold. On the date of sale, the house had a cost basis of $200,000. Within six months, Sanders purchased a new house for $600,000. What amount of gain should Sanders recognize from the sale of the residence g Which excerpt from My Lord Bag of Rice best reflects the historical fact that Hidesato was a brave warrior? A.At first Hidesato could not help feeling alarmed at the sight of this horrible reptile lying in his path. B.On turning back he was much surprised to see that the monster dragon had entirely disappeared. C.Hidesato showed not the least sign of fear. He tried to calm the Dragon King. Don't be afraid. I shall surely kill the centipede. D.The Dragon King did as he was bid, and the warrior noticed that he had only three arrows left in his quiver.